Ist der Impulsoperator ein Postulat?

Ich habe die Postulate des QM studiert und gesehen, wie man daraus wichtige Ideen ableiten kann. Eine Sache, die ich daraus jedoch nicht ableiten konnte, ist die Identität des Impulsoperators.

Der Einfachheit halber denke ich nur an keine relativistischen Effekte, keinen Spin, keine zeitabhängigen Potentiale und eine räumliche Dimension. Ich gehe auch davon aus, dass der Positionsoperator einfach multipliziert wird mit x , wie in, ich bin im Positionsraum. Der Hamilton-Operator ist also H = 2 2 m 2 + v .

Ich weiß, dass der Impulsoperator ist p = ich x .

Aber wie komme ich von den Postulaten dorthin? Ich weiß, dass es sinnvoll ist, da daraus das Ehrenfest-Theorem, die De-Broglie-Wellenlängenhypothese, die Heisenbergsche Unschärferelation (z x und p ), wobei der Impulsoperator der Generator des Übersetzungsoperators ist, und möglicherweise viele andere wünschenswerte Theoreme und Korrelationen mit dem klassischen Impuls.

Aber keines davon sind Postulate (zumindest nicht in den verschiedenen Formalismen, denen ich begegnet bin), also können Sie nicht ableiten p = ich x von ihnen. Vielmehr sind sie Folgen davon. Sie müssen den Operator vorher kennen, um zu sehen, dass er korrekt ist. Ja, das ist nur Semantik, aber das ist für mich der Kernpunkt:

Unabhängig davon, wie viel Sinn es macht, ist die Identität p = ich x (unter den von mir gemachten Annahmen) ein Postulat, das heißt, Sie können es nicht aus anderen Postulaten ableiten, oder können es tatsächlich von ihnen erhalten werden? Und im letzteren Fall, könnten Sie mir zeigen, wie?

Hinweis: Ich weiß, dass es viele verschiedene und gleichwertige Postulate für QM gibt. Aber in keiner, die ich gesehen habe, haben sie es als Postulat bezeichnet oder richtig abgeleitet.

Mögliche Duplikate: physical.stackexchange.com/q/45248/2451 , Physics.stackexchange.com /q/80357/2451 , Physics.StackExchange.com/q/53252/2451 , Physics.StackExchange.com/q/ 70203 / 2451 und Links darin.

Antworten (7)

Erstens ist es etwas holprig, so etwas zu schreiben:

P ^ = ich / x .
Es ist strenger zu schreiben:
x | P ^ | ϕ = ich x x | ϕ ,
und es sollte als Impulsoperator in räumlicher Darstellung interpretiert werden.

Ableitungen:

Die physikalische Bedeutung hinter Impuls ist: 1. Es ist die Erhaltungsgröße, die der räumlichen Translationssymmetrie entspricht. 2. Wegen 1 ist der Impulsoperator (hermitesch) der Generator des räumlichen Translationsoperators (unitär).

In Bezug auf Gleichungen:

Definieren Sie den räumlichen Übersetzungsoperator D ( a ) st

C | x + a = D ( a ) | x ,
und:
D ( a ) = e ich a p ^ /

Ich nehme an, Sie haben kein Problem damit, dies abzuleiten.

Bitte beachten Sie, dass dies nur von der Quantisierungsbedingung abhängt [ x , p ] = ich , was eines der Postulate der Quantenmechanik ist.

Nehmen Sie einen willkürlichen Zustand an | ϕ und bewerben D ( a ) darauf:

D ( a ) | ϕ = D ( a ) | ϕ | x x | d x
Variablenänderung, RHS =
C | x x a | ϕ d x

Nehmen a 0 , an RHS anschließen:

ϕ ( x a ) = ϕ ( x ) a x ϕ ( x )
und an LHS:
D ( a ) = 1 ich a p ^ /

du kannst dich erholen x | P ^ | ϕ = ich x x | ϕ

Natürlich haben Sie Recht damit, dass der Ausdruck eines Operators von der gewählten Darstellung abhängt, aber wenn ich diese Idee vorstelle, dann für Studenten, die ihre erste Begegnung mit QM haben. Im Moment ist das SE in einer Positionsdarstellung für sie Quantenmechanik. Für den Rest sehe ich keinen sinnvollen Unterschied zwischen der Aussage, ich werde annehmen, dass "die Form des Impulsoperators dies ist ", und der Aussage, "der Kommutator". [ x , p ] ist das ".
Vielen Dank! Die Antwort auf meine Frage wäre also ungefähr so: Definieren Sie zunächst Momentum in QM. Die Definition ist 1., und daraus erhalten Sie 2., was bedeutet, "der P-Operator erzeugt den Übersetzungsoperator". Verwenden Sie zweitens ein Postulat, [x,p] = iℏ, um zu zeigen, dass die obige Bedingung verlangt, dass ⟨x|P^|ϕ⟩=−iℏ∂⟨x|ϕ⟩/∂x. Am Ende GIBT es also ein Postulat bezüglich der Form von p.
@dmkee du hast recht!
Es tut mir leid, @HanaKaze, aber ich denke nicht, dass es logisch ist, zu sagen „Lass es mich jetzt für dich ableiten“ und dann eine Reihe von Annahmen über den Rahmen der Quantenmechanik zu treffen. Die klassische Mechanik sagt es nicht [ x , p ] = ich , und tatsächlich haben Sie dies in Ihrer Antwort erwähnt, sodass nichts "abgeleitet" wird. Dies sind alles Motivationen und schöne, um es gelinde auszudrücken. Lesen Sie meine Antwort unten.
Ich komme nicht umhin zu denken, dass der Teil „Zunächst ist es völlig falsch, so etwas zu schreiben wie P = ich / x “ ist selbst völlig falsch. Es ist üblich, im Hilbert-Raum zu arbeiten H = L 2 ( R ) von Wellenfunktionen in der x-Darstellung. Daran ist nichts falsch . Im Gegenteil, man könnte argumentieren, dass das Verlassen auf „Position Eigenkets | x ” Etwas zu definieren ist falsch, da der Positionsoperator als selbstadjungierter Operator auf H hat keine Eigenvektoren.
@m93a Ich stimme dir zu, dass ich nicht sagen sollte, dass etwas völlig falsch ist. Ich habe volles Vertrauen, dass Physiker verstehen, was das bedeutet, nur zu faul, es rigoros zu schreiben. Zweitens, Ihr Kommentar „man könnte argumentieren, dass es falsch ist, sich auf „Position Eigenkets ∣∣𝑥⟩“ zu verlassen, um etwas zu definieren“, ist genau das, was ich in meiner Antwort zu demonstrieren versucht habe, aus diesem Grund, lose geschrieben P = −𝑖ℏ∂/∂ 𝑥 ist keine so gute Praxis, auch wenn Sie wissen, dass es unter räumlicher Darstellung so aussieht.
Sie wissen, dass der vorherige Kommentar vor 5 Jahren geschrieben wurde?
@ziv Ich tbh, ich habe diesen Kommentar aus meiner Benachrichtigung gesehen und darauf geantwortet .... habe nicht einmal seinen Zeitstempel erkannt ....
@HanaKaze Ihr Beitrag ist voll von Dingen, die mathematisch schlecht definiert sind. Es ist nie „rigoroser“, etwas damit zu schreiben x | , wie du sagst. Der Betreiber ich d d x an L 2 ( R ) ist vollkommen rigoros ( Quelle ) und wird ständig von mathematischen Physikern verwendet. Der Ausdruck d d x x | ϕ ist undefiniert, es sei denn, Sie geben ein Rigging des Hilbert-Raums an ( Quelle ). Dein Beitrag ist einfach falsch.
Sie verwenden das Wort „streng“ in Bezug auf QM, ohne die strenge Formulierung von QM wirklich zu verstehen. Bitte tu das nicht, das führt andere in die Irre. Sie erkennen die Nichtexistenz von Eigenvektoren des Positionsoperators, aber Sie verwenden | x in formalen Ausdrücken, als ob sie existierten. Das ist keine Strenge. Noch lange nicht. An formalen Ableitungen ist a priori nichts auszusetzen, aber nennen Sie sie um Himmels willen nicht streng . Und nennen Sie nicht jeden, der QM anders macht als Sie, falsch (oder ruppig ).
@m93a danke für deine Kommentare, du bist genau der Grund, warum ich die Wissenschaft verlassen habe. Fühlen Sie sich frei, die Antwort abzulehnen oder zu löschen oder zu bearbeiten.
Liebe @HanaKaze, es tut mir wirklich leid, dass ich dir ein schlechtes Gewissen gemacht habe, ich hätte meine Kommentare anders formulieren sollen. Ich habe nicht die Privilegien, die Beiträge anderer zu bearbeiten. Ich habe eine Bearbeitung vorgeschlagen , die den Kern Ihrer Ableitung beibehält, aber den Wortlaut so ändert, dass er afaik viel präziser ist. Die Bearbeitung wurde von Mods abgelehnt, ich kenne den Grund nicht, es hat keine Diskussion stattgefunden. Daher ist der einzige Ort, an dem ich Ihren Beitrag korrigieren oder diskutieren kann, der Kommentarbereich.

Es gibt keine Ableitung, aber ein heuristisches Argument.

Nehmen wir an, wir schreiben das Jahr 1926 und Derby hat uns gerade herausgefordert, ihm die Wellengleichung zu zeigen, die zu den „Wellen“ von de Broglie gehört (so wie er Schrödinger herausgefordert hat). Das heißt, wir arbeiten an einer Wellengleichung. Die Lösungen sollten folgende Form haben (in einer Dimension)

Ψ ( x , t ) = EIN e ich ( k x ω t )
wo k = 2 π / λ ist die Wellenzahl und ω = 2 π / T ist die Winkelfrequenz.

Wir wollen auch

p = h / λ = k E = h f = ω
den Ad-hoc-Annahmen von de Broglie und Plank zuzustimmen, die funktionieren .

Wir könnten bemerken (wie ich annehme, dass Schrödinger es getan hat), dass die räumlichen und zeitlichen Ableitungen, die normalerweise in einer Wellengleichung erscheinen, uns Faktoren von liefern k und ω bzw. (mit einigen unbequemen Faktoren von ich herumhängen, aber damit müssen wir leben.). Das heißt, wir haben uns gerade entschieden, mitzumachen

p ich x E ich t

Von da an ist es nur noch eine Sache, dies für ein Teilchen zu sagen, das sich in einem Potential bewegt v die Gesamtenergie (in vielen Fällen hamiltonisch) ist

E = T + v = p 2 2 m + v ,
Wenn wir dies als eine Ableitung in Bezug auf die Zeit und zwei Ableitungen in Bezug auf den Raum betrachten und dann die Konstanten festlegen, können wir zu folgendem gelangen
(TDSE) [ 2 2 m 2 x 2 + v ( x ) ] Ψ ( x , t ) = ich t Ψ ( x , t )

Ich möchte wiederholen, dass dies in keiner Weise ein Beweis ist. Es ist eine Art erweitertes Plausibilitätsargument. Und eines, das die Aussetzung des Unglaubens ziemlich strapaziert, außer dass es funktioniert.


Ich habe eine sorgfältig konstruierte Version dieses Arguments, die ich meinen modernen Physikstudenten gebe, und Variationen können an vielen Stellen gefunden werden, die vor meiner Version liegen.

Es gibt eine Ableitung unter Verwendung des Satzes von Noether und der quasi-klassischen Grenze, vgl. Landau QM Abschnitt. 15: books.google.co.uk/…
@bolbteppa Die Sache ist, dass die quasi-klassische Grenze ein Verfahren ist, das gerechtfertigt ist, weil es funktioniert (dh ein Postulat), und nicht etwas, das sich unaufhaltsam aus der Mathematik ergibt. Wir müssen QM auf einer Reihe von Annahmen aufbauen. Das OP fragt, ob die Bedeutung von p ^ in der Positionsdarstellung ist eine davon und Sie sagen "nein, ich kann eine andere Annahme verwenden". Was wahr ist, aber dem Problem ausweicht, weil die beiden Annahmen einander äquivalent sind.
Wie ich bereits sagte, verstehe ich, warum die Form von p Sinn macht. Aber am Ende scheint es, als müsste man es direkt oder indirekt als Postulat angeben. Was nicht abgeleitet werden kann, muss postuliert werden. So sehe ich die Dinge zumindest.

Die Tatsache, dass P ^ ich / x in der Positionsbasis ist weder ableitbar noch ein Postulat, weil es nicht immer wahr ist. Die kanonische Kommutierungsrelation [ X ^ , P ^ ] = ich ich ^ wird im Allgemeinen als Postulat genommen, aber auch dann, wenn Sie die Darstellung wählen X ^ x in der Positionsbasis, dann erlaubt der CCR unendlich viele Darstellungen von P ^ des Formulars P ^ ( ich / x ) + f ( x ) für jede Funktion f ( x ) . Die Wahl der Darstellung entspricht einer Eichwahl für die Wellenfunktion und betrifft keine physikalisch beobachtbaren Größen. Siehe Übung 7.4.9 auf den Seiten. 213-214 von Shankar zur weiteren Diskussion.

Ich weiß, dass der Impulsoperator P = -iℏ ∂/∂x ist.

Allerdings ist es der Impulsoperator in der Ortsbasis . Der Impulsoperator in der Impulsbasis ist P = p in Analogie zum Positionsoperator in der Positionsbasis ist X = x .

(In starker Anlehnung an Brian Hatfields „Quantum Field Theory of Point Particles and Strings“)

Der Schlüssel ist, mit der Kommutierungsbeziehung zu beginnen

[ X , P ] = ich

Wenn | x bezeichnet dann einen Ortseigenzustand

X | x = x | x

und

x | X | x ' = x δ ( x x ' )

das heißt, dass der Betreiber X ist diagonal in der Positionsbasis. Wir suchen

x | P | x '

Seit

[ x , x ] = 1

Daraus folgt, dass der Betreiber ich x dient als Repräsentation von P auf dieser Basis und damit

x | P | x ' = ich x δ ( x x ' )


Wenn der Hamilton-Operator ist

H = P 2 2 m + v ( X )

dann

x | H | x ' = ( 2 2 m 2 x 2 + v ( x ) ) δ ( x x ' )

Nun ist die Schrödinger-Gleichung

ich t x | ψ ( t ) = x | H | ψ ( t )

Einfügen der Identität

1 = d x ' | x ' x ' |

Erträge

ich t x | ψ ( t ) = ich t ψ ( x , t ) = d x ' x | H | x ' x ' | ψ ( t ) = d x ' x | H | x ' ψ ( x ' , t )

und schließlich, unter Verwendung des Ergebnisses vom Anfang dieses Abschnitts,

ich t ψ ( x , t ) = d x ' ( 2 2 m 2 x 2 + v ( x ) ) δ ( x x ' ) ψ ( x ' , t ) = ( 2 2 m 2 x 2 + v ( x ) ) ψ ( x , t )

Sie haben die abstrakte Algebra:

[ x , p ] = ich

als Postulat. Es kommt entweder von der üblichen Poisson-Klammer, die zur Kommutatorregel geht, oder es wird nur abstrakt als Definition festgelegt.

Wie auch immer, Sie können nach Darstellungen dieser Algebra suchen. Das erste, was zu sehen ist, ist, dass es keine endlichdimensionalen Darstellungen (auch bekannt als Matrizen) gibt. Ein absurder Beweis geht davon aus, dass es möglich ist, und dann sollte man die Vertauschungsbeziehung verfolgen. Es ist offensichtlich, dass Sie bekommen

1 = 0 .

Das zweite, was Sie sehen sollten: Dies ist eine infinitesimale Übersetzung. In der Tat, bedenke a also ein infinitesimaler Parameter

δ x = [ x , a p ] = ich a ,

das ist die übliche infinitesimale Übersetzung, die man erwarten würde. Sie werden natürlich dazu gebracht, darüber nachzudenken a p als Generator von Übersetzungen.

Leider ist die Klassifizierung von Darstellungen unendlichdimensionaler Algebren ein heikles Thema. Ich weise Sie auf das Stone-von-Neumann-Theorem hin .

Das Beste, was ich tun kann, ist, die übliche Darstellung zu motivieren. Und es ist eigentlich nicht so schwer, weil wir nur die Diffeomorphismus-Algebra (denken Sie daran, sie sollte unendlich dimensional sein) verlassen haben, wo x und p soll auf Funktionen wirken.

Gegeben sei eine Funktion von x, genannt ψ ( x ) , kann eine Übersetzung durch die Taylor-Reihe erhalten werden:

ψ ( x + a ) = ψ ( x ) + a ψ ' ( x ) + a 2 2 ψ ( x ) + = exp ( a d d x ) ψ ( x ) ,

und da hast du es: p = ich d d x . Die Algebra wird dann durch die übliche Vektorfeldalgebra realisiert :

L p x = ich .

Sie sehen, Derivate erzeugen immer Übersetzungen. Die Quantenmechanik fordert Sie auf, sie Schwung zu nennen.

Ich überlasse es Ihnen, herauszufinden, was passiert wäre, wenn ich mich entschieden hätte, auf Funktionen von zu reagieren p .

Eine andere Antwort, die darauf hindeutet, dass es ein Postulat speziell für die Form von p gibt (in Bezug auf x). Vielen Dank. Ich schätze, die Lehrbücher, die ich gelesen habe, waren nicht so gründlich.
@JuanPerez Nein, warum würdest du das sagen? Es ist eine Repräsentation. Inhalt der Theorie sind Zustände und Operatoren, bei denen die Norm Wahrscheinlichkeit angibt. Eine Repräsentation ist nur eine Möglichkeit, diese Dinge darzustellen. Es ändert nichts am physischen Inhalt! Es ist auch nicht notwendig, es ist nur bequem.
Sie sagten gerade, dass Sie die abstrakte Algebra [x,p] = iℏ als Postulat nehmen. Was ich sagen wollte, ist, dass dieses Postulat speziell die Beziehung zwischen x und p angibt und dass Sie ohne es keinen funktionierenden Ausdruck des p-Operators erhalten könnten (der, wie Sie sagen, von der gewählten Darstellung abhängt).
Leider glaube ich nicht, dass ich mich klar ausgedrückt habe. Es ist kein zusätzliches Postulat erforderlich, um P = d/dx zu erhalten. Alles, was [x,P]=i erfüllt, ist gültig. Und Sie können diese Darstellungen ähnlich schreiben, wie Sie die Pauli-Matrizen nur aus der Drehimpulsalgebra erhalten. All diese Antworten zeigen Ihnen genau dasselbe.
Entschuldigung, dass ich immer wieder antworte, aber ich bin etwas verwirrt. Ich verstehe jetzt, dass P = -iℏd/dx kein Postulat ist. Aber [x,p] = iℏ ist eins, richtig? (Wenn es kein Postulat, sondern eine Wahl ist, könnte ich etwas anderes wählen, z. B. [x,p] = 0 oder [x,p] = 78, und gleichwertige Ergebnisse erhalten?)
Zwei Stücke antworten. 1) Bitte sagen Sie nicht, dass es Ihnen leid tut. Fragen sind erwünscht. Zu deiner Frage: Irgendwo muss man ja anfangen. Inzwischen ist Ihnen klar, dass das Wichtige ist [x,p] = eine Zahl ungleich Null, unabhängig von p und x. Jede Zahl ist in Ordnung, weil ich sie absorbieren und x und p neu definieren kann, um das übliche Zeug zu erhalten (niemand hat Ihnen gesagt, dass sie direkt klassischen Dingen entsprechen sollten). Der nicht triviale Teil besteht darin, eine Beziehung zu motivieren [ x , p ] 0 .
2) Jetzt sollten Sie irgendwo anfangen, um dies zu erhalten. Die Leute hier scheinen Symmetrie-Argumente zu mögen. Aber man hätte zum Beispiel von der Unschärferelation ausgehen können. Jede physikalische Sache, die Ihnen einfällt, die Sie dazu bringt, über diese Algebra nachzudenken, reicht aus.
Ja. Irgendwo muss man anfangen . Jedes davon kann das relevante Postulat sein, aber eines davon muss es sein. Du kannst sie nicht einfach aus den anderen Postulaten ableiten .

Lassen Sie mich als kurze Ergänzung zu den obigen Antworten wiederholen, dass keine der Quantenmechaniken von irgendeiner vorhergehenden Theorie „abgeleitet“ ist. Ja, es gibt viele Korrespondenzen, die ziemlich auffallend sind – kanonische Quantisierung, geometrische Quantisierung, Aktionswellen in der Hamilton-Jacobi-Theorie, Erweiterung der DeBroglie-Dispersionsbeziehung (worüber @dmckee gesprochen hat) usw. – und viele Leute nutzen diese, um die zu motivieren Entwicklung der Quantenmechanik aus Sicht der klassischen Physik. Aber am Ende des Tages ist die Quantenmechanik die grundlegendere Theorie, also wird sie postuliert (sie nennen sie aus einem bestimmten Grund "Postulate der QM").

Eine andere Sichtweise ist, dass die Rückgewinnung der Quantenmechanik aus der klassischen Physik kein gut gestelltes Problem ist. Wenn man die "klassische Grenze" der Quantenmechanik nimmt, gehen Informationen verloren, daher ist es nicht sinnvoll, die Quantenmechanik aus der klassischen Physik in ihrer strengen Definition "abzuleiten".

Diese Botschaft ist moralisch identisch mit dem, was Feynman in diesem populären Video betont .

Es wird festgestellt, dass "keine der Quantenmechaniken von irgendeiner vorhergehenden Theorie "abgeleitet" ist". Das ist nicht richtig. Die Quantenmechanik ist in der klassischen Feldtheorie begründet.
Könnten Sie "gegründet" definieren?

Der adäquatere "fundamentale" Weg besteht darin, ihn vom Kommutator abzuleiten , der Ihnen im Grunde sagt, wie der Informationskompromiss zwischen Position und Impuls funktioniert, was das Herzstück der Lektion der Quantenmechanik ist: Das Universum enthält ebenso eine Grenze des Informationsgehalts es hat eine Informationsgeschwindigkeitsbegrenzung. Beachten Sie, dass der Impulsoperator nur in Bezug auf die Position so aussieht , also setzt dies in gewissem Sinne auch voraus, dass wir auch die Position definiert haben.

Es ist eine empirische Tatsache, die durch viele wiederholte Versuche und Fehlschläge zu einem obszönen Vertrauensniveau etabliert wurde, dass es unmöglich ist, mehr Informationen von einem System über seine Position und seinen Impuls zusammen abzufragen, als durch den Grenzwert gegeben sind

H x + H p 1 + lg ( π )

ausgedrückt durch die Shannon-Entropie (beachten Sie, dass die Grenze davon abhängt, welche Einheiten Sie verwenden; technisch gesehen ist die Entropie relativ zu einer Skala) oder grober (nicht so stark, dh es gibt Fälle, in denen die untere Beziehung gilt, aber nicht die obere , und sie sind keine physikalisch gültigen Fälle, z. B. die Summe von zwei ausreichend weit voneinander entfernten Delta-Funktionen sowohl im Positions- als auch im Momentenraum) und typischerweise angegeben als

Δ x Δ p 1 2

.

In der auf linearer Algebra basierenden Sprache, die die Quantentheorie bereitstellt, sind das die Operatoren x ^ und p ^ befriedigen muss

[ x ^ , p ^ ] = ich

Wenn Sie zwei beliebige Operatoren haben, die dies erfüllen, können Sie dies zeigen, wenn Sie die Eigenzustände von einem von ihnen als Basis verwenden , indiziert durch x unten, dann muss der andere die Form haben

p ^ = ich x

. Sie müssen also nicht einmal versuchen, herauszufinden, was die Grundlage dafür ist x ^ oder p ^ ist, nehmen Sie einfach an, dass es existiert, und leiten Sie entsprechend ab. Wenn Sie eine andere Basis als die von jemand anderem verwendet haben, funktioniert die Mathematik immer noch auf die gleiche Weise.

Daher wäre der Impulsoperator selbst nicht direkt ein Postulat. Vielmehr sollten wir als Teil der Beschreibung eines einzelnen Teilchens die Vertauschungsbeziehung zwischen Ort und Impuls angeben, und das definiert beide auf einmal.